Which one of the following could be the tour's schedule, with the four cities included in the tour listed in the orde...

hfatima1 on May 22, 2020

Game Setup

Can someone please show me the set up for this game?

Reply
Create a free account to read and take part in forum discussions.

Already have an account? log in

Irina on May 23, 2020

Hi @hfatima1,

This is a linear game that requires us to determine the order of visits to four out of six cities - H J M O S T. Each city is visited only once. We have the following rules:

(1) H and T must be included in the tour but they cannot be visited consequently.

This rule tells us that H and T must appear on our schedule, and both appear exactly once but not next to each other, there must be at least one or two other cities between them.

___ ___ ___ ___ H T

(2) if O is included in the tour, S cannot be.
Conversely, we can infer that if S is included in the tour then O is not. These two visits are mutually exclusive, meaning only one of them could be visited or neither, which leaves us with two possible scenarios for a complete list of visited cities:

H T S/O J/M
H T J M

O -> ~S
S -> ~O

(3) If J is visited, it must be third.

J -> 3d

(4) If both J & M are included they must be visited consecutively.
We can see that the only way both J and M could be included is if both S & O are excluded, meaning the complete list of cities visited in that scenario is:

H T J M

Since JM must be consecutive and H T cannot be consecutive, we can infer that H and T must be in positions 1 or 4 and MJ in positions 2 and 3 respectively (remember J must always be 3d)

H/T M J H/T

Putting all the rules together we have the following setup

(1) H/T M J H/T
(2) ___ ___ __ __ H T S/O M/J

~HT (not consecutive)
J = 3d

The question asks us which of the following could be the tour's schedule:

(A) J T S H

Incorrect. J must always be third.

(B) M T J H
Incorrect. MJ must be consecutive when they are included

(C) O H S T

Incorrect. O and S are mutually exclusive.

(D) S T M H
Correct. This order complies with all the rules.

(E) T M J S

Incorrect. H and T must always be included.

Question 2 is asking if S is visited 4th which of the following must be true?

If S is visited 4th we know that we are in scenario 2, and H/T must be visited 1st and 3, thus J cannot be visited (it must always be 3d), and the only remaining city - M must be visited 2d.

H/T M H/T S

Let me know if this makes sense and if you have any other questions.